Which one of the following must be false?

ahambsch16 on July 20, 2020

answer

can an explanation of the correct answer be given

Reply
Create a free account to read and take part in forum discussions.

Already have an account? log in

Emil-Kunkin on April 20 at 08:13PM

For a must be false we are trying to break this game. Here, the two easiest ways would be to break apart JG, or to force GK to be far apart.

I would first look for something that breaks up our block of JG, and the easiest way to break that would be to put one thing in the middle day that is neither j nor g. Thankfully, we have an answer choice that does just that in the form of c